Partial derivative using only function notation

Click For Summary
To find the partial derivative of q(x, y, e(x, y, u)) with respect to x, the chain rule is applicable since e is a function of x, y, and u. The correct approach involves differentiating q with respect to e and then multiplying by the derivative of e with respect to x. The user expresses confusion about applying the product rule, clarifying that this situation involves a function within a function rather than a product of two functions. The discussion highlights the need to treat x, y, and u as independent variables when differentiating. The solution requires careful application of the chain rule to navigate the nested function structure.
Bman12345
Messages
2
Reaction score
0

Homework Statement


I need to find the partial derivative of the following, with respect to x


q(x,y,e(x,y,u))
where e(x,y,u) is a function


Homework Equations





The Attempt at a Solution


Well, the problem is I don't have a clue how to solve using just the function notation - I'm used to doing it to an actual fuction (if that makes sence)

so I tried doing the chain rule,
(I will use d as I don't know how to get the patial derivative symbol)

\frac{d q(x,y,e(x,y,u)}{d e(x,y,u)} \times \frac{d e(x,y,u)}{d x}

However I do not think this is right. I don't think I use the product rule as it seems to be a function within a function, not two functions times together.

So yeah, what should be an easy problem has me stumped!
 
Physics news on Phys.org
Use the chain rule, but it should be a sum.

[f(u(x,y,x),v(x,y,x),w(x,y,x))]x=f(1,0,0) ux+f(0,1,0) vx+f(0,0,1) wx=fu ux+fv vx+fw wx

where raised (a,b,c) means differentiate slot 1 a times, slot 2 b times, and slot 3 c times
I assume x,y,u are independent variables.
 
Last edited:
Question: A clock's minute hand has length 4 and its hour hand has length 3. What is the distance between the tips at the moment when it is increasing most rapidly?(Putnam Exam Question) Answer: Making assumption that both the hands moves at constant angular velocities, the answer is ## \sqrt{7} .## But don't you think this assumption is somewhat doubtful and wrong?

Similar threads

Replies
26
Views
3K
  • · Replies 3 ·
Replies
3
Views
2K
  • · Replies 2 ·
Replies
2
Views
1K
Replies
5
Views
2K
  • · Replies 10 ·
Replies
10
Views
2K
  • · Replies 6 ·
Replies
6
Views
2K
  • · Replies 4 ·
Replies
4
Views
1K
Replies
4
Views
1K
  • · Replies 4 ·
Replies
4
Views
2K
Replies
9
Views
2K